Gastrointestinal Tract 1 (1-1 - 6-20)

General

1-1 (100G42) Which combination of gastrointestinal hormones and their actions is correct?

a) Gastrin - Inhibits gastric secretion

b) Glucagon - Promotes pancreatic fluid secretion

c) Cholecystokinin - Suppresses bile secretion

d) Secretin - Promotes bile secretion

e) Somatostatin - Stimulates pancreatic fluid secretion

 

The correct answer is:

d) Secretin - Promotes bile secretion

Explanation:

  • Gastrin actually stimulates gastric acid secretion, not inhibits it.
  • Glucagon mainly affects blood glucose levels and does not directly promote pancreatic fluid secretion in the context mentioned.
  • Cholecystokinin (CCK) does not suppress bile secretion; it actually promotes bile secretion by contracting the gallbladder and stimulates the secretion of pancreatic enzyme-rich juice.
  • Secretin indeed promotes bile secretion (choleretic) and also stimulates the pancreas to release bicarbonate-rich pancreatic juice.
  • Somatostatin inhibits the release of several hormones and slows down gastric emptying and secretion, not stimulates pancreatic fluid secretion.

So, the corrected statement with accurate hormone actions is: d) Secretin - Promotes bile secretion

1-2 (101B35) Which combination of blood vessel branching is correct?

a) Celiac artery branches into the inferior phrenic artery.

b) Common hepatic artery branches into the gastroduodenal artery.

c) Splenic artery branches into the right gastric artery.

d) Superior mesenteric artery branches into the left colonic artery.

e) Inferior mesenteric artery branches into the ileocolic artery.

 

The correct answer is:

b) Common hepatic artery branches into the gastroduodenal artery.

Explanation: 

  • The celiac artery does not branch into the inferior phrenic artery; the inferior phrenic arteries typically arise directly from the aorta.
  • The common hepatic artery indeed branches into the gastroduodenal artery, making option b correct.
  • The splenic artery does not branch into the right gastric artery; the right gastric artery usually branches from the common hepatic artery.
  • The superior mesenteric artery does not directly branch into the left colonic artery; the superior mesenteric artery supplies the midgut, including parts of the colon, but the left colonic artery typically arises from the inferior mesenteric artery.
  • The inferior mesenteric artery does branch into arteries supplying the lower colon, but the term "ileocolic artery" is more accurately associated with branching from the superior mesenteric artery as it supplies the ileum and the beginning of the colon.

Thus, the correct statement, with accurate anatomical relationships, is: b) Common hepatic artery branches into the gastroduodenal artery.

1-3 (102B40) Which one does not have a serous membrane?

a) Esophagus

b) Stomach

c) Duodenum

d) Jejunum

e) Transverse colon

 

The correct answer is:

a) Esophagus

Explanation:

  • The esophagus does not have a serous membrane. Instead, it is surrounded by connective tissue without a serous layer in most of its extent, especially in the thoracic part.
  • The stomachduodenumjejunum, and transverse colon are all parts of the gastrointestinal tract that are covered by the peritoneum, a serous membrane, in at least some portion of their extents. The duodenum has parts that are retroperitoneal (meaning they are covered by peritoneum only on the anterior side), but still, it is associated with a serous covering where applicable.

1-4 (102G2) Which sign has the highest positive likelihood ratio for detecting peritonitis?

a) Rovsing's sign

b) Murphy's sign

c) Muscular defense

d) Rebound tenderness

e) Board-like rigidity

 

The correct answer is:

e) Board-like rigidity

Explanation: 

  • Rovsing's sign is indicative of appendicitis, not directly peritonitis, and involves pain in the right lower quadrant upon palpation of the left lower quadrant.
  • Murphy's sign is a sign of cholecystitis (inflammation of the gallbladder), elicited by palpating the right upper quadrant, causing pain and a halt in inspiration.
  • Muscular defense (muscular guarding) refers to the involuntary tensing of abdominal muscles during palpation, suggesting peritoneal irritation.
  • Rebound tenderness involves pain upon the release of pressure on the abdomen, indicating peritoneal irritation.
  • Board-like rigidity is a term that describes the extreme stiffness of the abdominal muscles, a severe form of guarding, indicative of generalized peritonitis.

Of these, board-like rigidity (option e) is often considered a sign with a high positive likelihood ratio for detecting peritonitis, as it indicates severe, diffuse irritation of the peritoneum. However, the specific positive likelihood ratio can vary depending on the study and context, and clinical judgment should consider all signs in conjunction with the patient's symptoms and medical history.

1-5 (103G13) Which is not fixed in the retroperitoneum?

a) Kidney

b) Duodenum

c) Ascending colon

d) Transverse colon

e) Abdominal aorta

 

The correct answer is:

d) Transverse colon

Explanation:

  • The kidneyduodenumascending colon, and abdominal aorta are primarily retroperitoneal, meaning they are located behind the peritoneum in the abdominal cavity.
  • The transverse colon, however, is not fixed in the retroperitoneum; it is intraperitoneal, meaning it is encased in peritoneum and has a mesentery allowing for some mobility.

1-6 (104G27) Which three of the following statements about digestion and absorption are true?

a) Carbohydrates are absorbed in the upper small intestine.

b) Triglycerides are digested by lipase.

c) Proteins are digested with amylase.

d) Vitamin B12 is absorbed by binding to intrinsic factor.

e) Iron is absorbed in the terminal ileum.

 

The correct answers, based on standard physiological knowledge, are:

a) Carbohydrates are absorbed in the upper small intestine.

b) Triglycerides are digested by lipase.

d) Vitamin B12 is absorbed by binding to intrinsic factor.

Explanation:

  • Carbohydrates are indeed absorbed in the upper parts of the small intestine, primarily the duodenum and jejunum.
  • Triglycerides are digested by lipases, which are enzymes that break down fats into fatty acids and glycerol.
  • Proteins are not digested by amylase; they are digested by proteases. Amylase is an enzyme that digests carbohydrates, so option c is incorrect.
  • Vitamin B12 is absorbed in the ileum, but only after binding to the intrinsic factor, which is secreted by the stomach.
  • While iron is absorbed mainly in the duodenum and upper jejunum, not in the terminal ileum, making option e incorrect in this context. However, there can be some absorption in the terminal ileum under certain conditions, but it's not the primary site.

1-7 (105B20) Which of the muscles involved in bowel function is an involuntary muscle?

a) Internal anal sphincter

b) External anal sphincter

c) Puborectalis muscle

d) Pubococcygeus muscle

e) Iliococcygeus muscle

 

The correct answer is:

a) Internal anal sphincter

Explanation:

  • The internal anal sphincter is composed of smooth muscle, which is not under voluntary control, and therefore it is an involuntary muscle.
  • The external anal sphincterpuborectalis musclepubococcygeus muscle, and iliococcygeus muscle are all part of the pelvic floor and are composed of skeletal muscle, which is under voluntary control.

1-8 (105E6)  Which of the following veins does not flow into the portal vein?

a) Splenic vein

b) Left gastric vein

c) Internal iliac vein

d) Superior mesenteric vein

e) Inferior mesenteric vein

 

The correct answer is:

c) Internal iliac vein

Explanation:

  • The splenic veinleft gastric veinsuperior mesenteric vein, and inferior mesenteric vein are all tributaries of the portal vein, which carries blood to the liver from the intestines and spleen.
  • The internal iliac vein is not a tributary of the portal vein; it drains into the common iliac vein and then into the inferior vena cava, which carries blood back to the heart.

1-9 (106E33) Which of the following is incorrect during bowel movements?

a) Peristalsis of the large intestine occurs.

b) Rectal pressure increases.

c) The internal anal sphincter contracts.

d) The puborectalis muscle relaxes.

e) The anal canal and rectum are straightened.

 

The correct answer is:

c) The internal anal sphincter contracts.

Explanation:

  • During a bowel movement, peristalsis of the large intestine does indeed occur to move fecal matter towards the rectum.
  • Rectal pressure increases as fecal matter enters the rectum.
  • The internal anal sphincter actually relaxes (not contracts) to allow the passage of feces.
  • The puborectalis muscle, part of the pelvic floor, relaxes to straighten the anorectal angle, facilitating defecation.
  • The anal canal and rectum are straightened as part of the defecation process to allow the feces to pass.

1-10 (106G33) Which of the following is contained in oral rehydration solutions and is conjugated with glucose in the small intestine to promote water absorption?

a) Sodium (Na)

b) Potassium (K)

c) Chloride (Cl)

d) Calcium (Ca)

e) Magnesium (Mg)

 

The correct answer is:

a) Sodium (Na)

Explanation:

  • Oral rehydration solutions typically contain sodium in the form of sodium chloride or sodium citrate. Sodium is co-transported with glucose across the intestinal wall via the sodium-glucose linked transporter (SGLT1), which promotes the absorption of water into the bloodstream. This is the basis for oral rehydration therapy used in cases of dehydration, often due to diarrhea.

1-11 (106G36) Which two of the following increase gastric acid secretion?

a) Gastrin

b) Secretin

c) Somatostatin

d) Glucose-dependent insulinotropic peptide (GIP)

e) Histamine

 

The correct answers are:

a) Gastrin

e) Histamine

Explanation:

  • Gastrin is a hormone that stimulates the secretion of gastric acid by the parietal cells of the stomach.
  • Histamine binds to H2 receptors on gastric parietal cells, promoting increased secretion of gastric acid.

On the other hand:

  • Secretin actually inhibits gastric acid secretion and stimulates the production of bicarbonate in the pancreas.
  • Somatostatin also inhibits the secretion of gastric acid.
  • Glucose-dependent insulinotropic peptide (GIP) does not directly increase gastric acid secretion; it primarily induces insulin release after glucose intake.

1-12 (109E13) Which statement is correct regarding the dissection of the vascular structures in the epigastric region?

a) The Glisson capsule contains hepatic arteries, hepatic veins, and bile ducts.

b) The common hepatic artery and the common bile duct run together.

c) The splenic artery branches from the celiac artery.

d) The superior mesenteric vein and the inferior mesenteric vein merge to form the portal vein.

e) The superior mesenteric artery runs dorsal to the horizontal part of the duodenum to the ascending colon.

 

The correct answer is:

c) The splenic artery branches from the celiac artery.

Explanation:

  • The Glisson capsule, or Glisson's capsule, refers to the connective tissue sheath enclosing the hepatic triad of portal vein branches, hepatic artery branches, and bile ducts, not hepatic veins, making option a incorrect.
  • The common hepatic artery and the common bile duct do not run together; they are part of the hepatic portal triad but have separate paths, making option b incorrect.
  • The splenic artery does indeed branch from the celiac artery, making option c correct.
  • The superior mesenteric vein and the inferior mesenteric vein do merge, but they join with the splenic vein to form the portal vein, not directly with each other, which makes option d incorrect as phrased.
  • The superior mesenteric artery runs behind the pancreas and in front of the uncinate process of the pancreas, not dorsal to the horizontal duodenum, and it supplies the midgut, which extends to the proximal two-thirds of the transverse colon, not specifically to the ascending colon, making option e incorrect.

1-13 (110E4) Which substances are absorbed from the stomach?

a) Iron

b) Folic acid

c) Fatty acids

d) Ethanol

e) Glucose

 

The correct answer is:

d) Ethanol

Explanation:

  • The stomach can absorb small quantities of ethanol (alcohol) due to its simple molecular structure that allows it to diffuse through the gastric mucosa.
  • Iron is primarily absorbed in the duodenum and upper jejunum, not in the stomach.
  • Folic acid is absorbed in the upper part of the small intestine, mainly in the duodenum and jejunum.
  • Fatty acids are not absorbed in the stomach; they are absorbed in the small intestine after emulsification and digestion.
  • Glucose is absorbed in the small intestine, not in the stomach.

1-14 (110I17) Which blood levels increase with the administration of proton pump inhibitors?

a) Ghrelin

b) Gastrin

c) Somatostatin

d) Cholecystokinin

e) Glucagon-like peptide 1 (GLP-1)

 

The correct answer is:

b) Gastrin

Explanation:

  • Proton pump inhibitors (PPIs) work by reducing the production of stomach acid. When stomach acid is suppressed, the body may respond by increasing the levels of gastrin, a hormone that stimulates acid secretion, in an attempt to maintain acid balance. This is known as a feedback mechanism.

1-15 (111I28) Which two areas are more often affected by volvulus?

a) Stomach

b) Duodenum

c) Descending colon

d) Sigmoid colon

e) Rectum

 

The correct answers are:

a) Stomach

d) Sigmoid colon

Explanation:

  • Volvulus is a condition where a part of the intestine twists around itself and the mesentery that supports it, causing a bowel obstruction. It most commonly affects the sigmoid colon due to its mobility and the length of its mesentery.
  • Stomach (Gastric volvulus) is indeed a rare condition but can occur, involving the rotation of the stomach by more than 180 degrees, leading to a potential blockage of blood flow and food passage.
  • Sigmoid colon is the most common site for volvulus, particularly in adults, due to its length and mobility, which makes it prone to twisting on itself.

1-16 (113C12) Which of the following statements is true about the digestion and absorption functions of the digestive tract?

a) Obstructive jaundice does not affect stool color.

b) Digestion is not required for the absorption of proteins.

c) Dietary fiber does not affect the absorption of sugars.

d) Medium-chain fatty acids are transported to the lymphatic vessels.

e) Bile acids are required for the absorption of long-chain fatty acids.

 

The correct answer is:

e) Bile acids are required for the absorption of long-chain fatty acids.

Explanation:

  • Obstructive jaundice typically leads to pale stools due to the lack of bile reaching the intestine, making option a incorrect.
  • Digestion is necessary for the absorption of proteins, as proteins need to be broken down into amino acids or small peptides before they can be absorbed, making option b incorrect.
  • Dietary fiber can indeed affect the absorption of sugars by slowing down the digestive process and reducing the rate at which sugars are absorbed, making option c incorrect.
  • Medium-chain fatty acids are absorbed directly into the portal vein and transported to the liver via the bloodstream, not the lymphatic vessels, making option d incorrect.
  • Bile acids emulsify fats in the small intestine, which is necessary for the digestion and absorption of long-chain fatty acids, making option e correct.

Clinical

2-1 (106D56) A 76-year-old woman suddenly experienced severe abdominal pain after dinner and was brought to the hospital 2 hours later. She had no history of abdominal surgery. Her SpO2 was 94% on room air. The abdomen was slightly stiff, and tenderness and mild symptoms of peritoneal irritation were observed overall. She was diagnosed with a superior mesenteric artery embolism and underwent immediate open surgery. Which two of the following are expected to have impaired absorption postoperatively in this patient?

a) Glutamine

b) Bile acids

c) Sodium

d) Vitamin B12

e) Glucose

 

The correct answers are likely:

b) Bile acids d) Vitamin B12

Explanation:

  • Bile acids are essential for the emulsification and absorption of fats and fat-soluble vitamins. Impaired absorption may occur postoperatively, especially if there is damage or reduced blood flow to areas of the small intestine responsible for fat absorption.

  • Vitamin B12 absorption can be impaired if the surgery affected the terminal ileum, where Vitamin B12 is absorbed in conjunction with intrinsic factor. A superior mesenteric artery embolism can lead to ischemia in portions of the midgut, which may include the ileum.

Absorption of glutamine, sodium, and glucose could also be affected depending on the extent of the intestinal damage and the specific sections of the intestine that were compromised. However, bile acids and Vitamin B12 are more specifically absorbed in distinct areas of the small intestine that could be affected by ischemia due to a superior mesenteric artery embolism, leading to more notable absorption issues for fats, fat-soluble vitamins, and Vitamin B12.

2-2 (111D035) A 69-year-old woman presented to the hospital complaining of blood in her stool. Her medical history is noncontributory. A lower gastrointestinal endoscopy revealed a lesion located 20 cm from the anus on the oral side. Which artery is typically ligated at the root during radical surgery for this condition?

a) Superior mesenteric artery

b) Ileocolic artery

c) Middle colic artery

d) Inferior mesenteric artery

e) Internal iliac artery

The correct answer is:

d) Inferior mesenteric artery

Explanation:

  • The inferior mesenteric artery supplies blood to the distal part of the colon, which includes the descending colon, sigmoid colon, and part of the rectum. A lesion found 20 cm from the anus suggests that it is in the sigmoid colon or the rectal area, which are supplied by the branches of the inferior mesenteric artery.
  • The superior mesenteric artery supplies the midgut, which includes the jejunum, ileum, and ascending colon, and would not typically be ligated for a lesion in the sigmoid colon or rectum.
  • The ileocolic artery is a branch of the superior mesenteric artery that supplies the ileum and the beginning of the colon, and it would not be ligated for a lesion 20 cm from the anus.
  • The middle colic artery is a branch of the superior mesenteric artery that supplies the transverse colon, and it would not be ligated for a lesion in the sigmoid colon or rectum.
  • The internal iliac artery primarily supplies the pelvic organs and would not be ligated for a lesion in this part of the colon.

2-3 (112F54) A 54-year-old man presented to the hospital with a complaint of vomiting blood. He has had black stools for three days but did not seek medical attention until this morning when he vomited about one cup of blood, prompting him to visit the emergency room. His skin is moist and cool to the touch, and there is pallor in his limbs. The eyelid conjunctiva is mildly anemic, but there is no jaundice in the ocular conjunctiva. His abdomen is flat with tenderness in the epigastric region, but there is no muscular guarding.

What should be the next step in management?

a) Blood transfusion

b) Angiography

c) Open surgery

d) Upper gastrointestinal endoscopy

e) Intravenous proton pump inhibitor

 

The correct answer is:

d) Upper gastrointestinal endoscopy

Explanation:

  • An upper gastrointestinal endoscopy is the next appropriate step to diagnose the source of the bleeding. It allows direct visualization of the upper digestive tract, which includes the esophagus, stomach, and duodenum, where a common source of bleeding (such as an ulcer) can be identified and potentially treated.

  • While a blood transfusion might be necessary if the patient is hemodynamically unstable due to blood loss, the need for it would typically be determined after stabilization and assessment of the extent of blood loss.

  • Angiography could be considered if the bleeding is brisk and the source needs to be identified for possible embolization, but it is not usually the first step.

  • Open surgery would be premature without first identifying the source of bleeding through less invasive means.

  • An intravenous proton pump inhibitor could be part of the initial management to reduce further gastric bleeding, especially if an ulcer is suspected, but it does not replace the need for diagnostic endoscopy.

2-4 (114C41) A 59-year-old woman came to the hospital after a positive fecal occult blood test during a medical examination. A lower gastrointestinal endoscopy was performed, and ascending colon cancer was diagnosed. There were no apparent metastases on imaging tests such as a CT scan, and she underwent a right hemicolectomy. Her height is 156 cm, weight 48 kg, body temperature is 36.2°C, pulse rate is 72/min, blood pressure is 108/60 mmHg, and respiratory rate is 12 per minute. There are no abnormalities in the eyelid conjunctiva and ocular conjunctiva, heart sounds, and breath sounds are normal, the abdomen is flat and soft, and there is no palpability of the liver and spleen. Blood findings are: red blood cells 3.98 million, hemoglobin 12.5 g/dL, hematocrit 39%, leukocytes 4,900, platelets 140,000. Blood biochemical findings are: total protein 6.6 g/dL, albumin 3.8 g/dL, total bilirubin 0.8 mg/dL, AST 16 U/L, ALT 18 U/L, LDH 184 U/L (reference range 120-245), ALP 202 U/L (reference range 115-359), creatinine 1.0 mg/dL, sodium 141 mEq/L, potassium 4.0 mEq/L, chloride 101 mEq/L.

Which of the following is correct in perioperative management?

a) Fasting for 48 hours before surgery

b) Drinking water up to 3 hours before surgery

c) Intraoperative high-volume infusion

d) Bed rest for 3 days postoperatively

e) Fasting for 1 week postoperatively

 

The correct answer is:

b) Drinking water up to 3 hours before surgery

Explanation:

  • Current perioperative guidelines allow for the consumption of clear fluids, including water, up to 2-3 hours before surgery. This helps to reduce the patient's discomfort and thirst without increasing the risk of aspiration during anesthesia.

  • Fasting for 48 hours before surgery or for 1 week postoperatively is not typically recommended as it can delay recovery and increase the risk of complications such as infection and poor wound healing.

  • Intraoperative high-volume infusion is not universally correct; fluid management should be tailored to the patient's needs, and excessive infusion can lead to complications.

  • Prolonged bed rest postoperatively is also not recommended as it increases the risk of deep vein thrombosis (DVT) and other complications. Early mobilization is encouraged to promote circulation and reduce the risk of pulmonary complications.

2-5 (115A42) A 58-year-old man visited the hospital complaining of a sensation of incomplete bowel evacuation. He has been aware of this sensation for six months and sought medical attention because it persisted. His stools are pellet-like, and he has a bowel movement approximately every three days. Despite straining during each bowel movement, the sensation of incomplete evacuation remains. His medical history is unremarkable. Physical examination reveals a flat and soft abdomen with no tenderness. Digital rectal examination and lower gastrointestinal endoscopy show no abnormalities.

What is the appropriate management?

a) Rest instructions

b) Antimicrobial administration

c) Periodic enemas

d) Osmotic laxatives

e) Restriction of dietary fiber intake

 

The correct answer is:

d) Osmotic laxatives

Explanation:

  • The patient's symptoms suggest functional constipation, characterized by infrequent stools and a sensation of incomplete evacuation.

  • Osmotic laxatives are often used to treat constipation as they help to soften the stool and improve bowel movement frequency.

  • There is no indication for rest, antimicrobial administration, or dietary fiber restriction based on the symptoms described.

  • Periodic enemas can be used in some cases of constipation, but they are typically not the first line of treatment and are usually reserved for cases where other treatments have failed or are not suitable.

  • Restricting dietary fiber intake is generally not recommended for constipation; in fact, a diet rich in fiber is usually advised to help promote regular bowel movements, unless there is a specific contraindication.

2-6 (116A38) A 62-year-old man has been diagnosed with descending colon cancer and is scheduled for a left hemicolectomy via laparotomy. At the age of 28, he was hospitalized for a month due to peritonitis following appendicitis. His height is 175 cm, weight 60 kg, body temperature is 36.2°C, pulse rate is 76/min, blood pressure is 120/70 mmHg, and respiratory rate is 14 per minute. There are no abnormalities in the eyelid or ocular conjunctiva, heart and breath sounds are normal, the abdomen is flat and soft, and the liver and spleen are not palpable. Surgical scars are present in the lower right abdomen and midline. Blood findings are: red blood cells 4.1 million, hemoglobin 13.8 g/dL, hematocrit 42%, leukocytes 5,200, and platelets 160,000. PT-INR is 1.0 (standard 0.9 to 1.1), APTT is 29.0 sec (reference control 32.2), and blood biochemical findings are: total protein 7.4 g/dL, albumin 4.0 g/dL, total bilirubin 0.8 mg/dL, AST 18 U/L, ALT 20 U/L, LDH 196 U/L (reference 120 to 245), ALP 102 U/L (reference 38 to 113), creatinine 0.8 mg/dL, Na 140 mEq/L, K 3.8 mEq/L, Cl 100 mEq/L, and CEA 5.2 ng/mL (standard 5 or less).

Which of the following is appropriate for the patient's perioperative management?

a) Allow solids intake up to 2 hours before surgery.

b) Bed rest is required for 3 days after surgery.

c) A liquid diet should be started one week after surgery.

d) Control the patient's body temperature to room temperature during surgery.

e) Perform analgesia with continuous epidural anesthesia.

 

The correct answer is:

e) Perform analgesia with continuous epidural anesthesia.

Explanation:

  • Continuous epidural anesthesia is often used for pain control in the perioperative period for abdominal surgeries. It can provide effective analgesia and may facilitate earlier mobilization.

  • It is not recommended to allow solids intake up to 2 hours before surgery; typically, patients are required to fast from solids for at least 6-8 hours before surgery to reduce the risk of aspiration during anesthesia.

  • Prolonged bed rest after surgery is not recommended as it can increase the risk of complications such as deep vein thrombosis (DVT) and pulmonary embolism (PE). Early mobilization is encouraged.

  • A liquid diet is typically started as soon as bowel function returns after surgery, which may be sooner than one week.

  • The patient's body temperature should be maintained within a normal range during surgery to prevent hypothermia, not necessarily at room temperature. Hypothermia during surgery can increase the risk of bleeding and infection.

2-7 (116C37) A 75-year-old woman presented to the hospital with a complaint of bloody stool. She has had occasional episodes of dark red bloody stools for the past six months, which improved spontaneously, so she did not seek immediate medical attention. Three days prior to the visit, she noticed blood in her stool again, prompting her to consult a doctor. On the day of the examination, her stools had returned to normal. Her medical history is unremarkable, and she is not currently on any oral medications. She is alert, with a body temperature of 36.8°C, a pulse rate of 84/min, blood pressure of 116/84 mmHg, and a respiratory rate of 16/min. SpO2 is 96% on room air. There is mild anemia noted in the eyelid conjunctiva. The abdomen is flat, soft, and non-tender. Digital rectal examination shows no abnormalities. Blood findings include erythrocytes at 3.45 million, Hb at 8.6 g/dL, hematocrit at 26%, leukocytes at 7,400, and platelets at 260,000. Blood biochemical findings are total protein at 6.8 g/dL, albumin at 3.6 g/dL, total bilirubin at 0.9 mg/dL, AST at 26 U/L, ALT at 27 U/L, LDH at 265 U/L (reference range 120-245), amylase at 65 U/L (reference range 37-160), urea nitrogen at 21 mg/dL, creatinine at 0.8 mg/dL, iron at 23 μg/dL, ferritin at 10 ng/mL (reference range 20-120), TIBC at 412 μg/dL (reference range 290-390), blood glucose at 101 mg/dL, sodium at 142 mEq/L, potassium at 4.6 mEq/L, and chloride at 99 mEq/L. CRP is 0.1 mg/dL. The fecal occult blood test was positive. Abdominography, CT, upper and lower gastrointestinal endoscopy were performed, but no lesions were found.

What is the appropriate test to perform next?

a) Small bowel enterography

b) Abdominal MRI

c) Abdominal angiography

d) Small bowel capsule endoscopy

e) Meckel's diverticulum scintigraphy

 

The correct answer is:

d) Small bowel capsule endoscopy

Explanation:

  • Given the patient's presentation with bloody stools and the fact that standard imaging and endoscopic evaluations have not identified a source of bleeding, a small bowel capsule endoscopy is a logical next step. This non-invasive test allows visualization of the small intestine, which is not easily accessible with traditional endoscopy.

  • Small bowel enterography could also be considered to visualize the small bowel, but it may not be as sensitive as capsule endoscopy for detecting small lesions.

  • An abdominal MRI could be useful for imaging soft tissue and other structures, but it is not the first choice for investigating obscure gastrointestinal bleeding.

  • Abdominal angiography is typically reserved for cases where there is active bleeding that might be amenable to embolization.

  • Meckel's diverticulum scintigraphy could be considered if there is suspicion of a bleeding Meckel's diverticulum, especially in younger patients, but it is less common in elderly patients. However, given the negative findings on other tests and the patient's age, it may be less likely compared to other causes of small bowel bleeding.

General

3-1 (103B30) Which of the following is the least likely cause of hematemesis?

a) Achalasia of the esophagus

b) Esophageal varices

c) Mallory-Weiss syndrome

d) Acute gastric mucosal lesions

e) Gastric cancer

 

The correct answer is:

a) Achalasia of the esophagus

Explanation:

  • Achalasia is a condition where the lower esophageal sphincter fails to relax properly, causing difficulty in passing food into the stomach. While it can lead to esophageal dilation and potential esophageal damage over time, it is not commonly associated with hematemesis (vomiting blood).

  • Esophageal varices, Mallory-Weiss syndrome (tears in the mucosa at the junction of the stomach and esophagus), acute gastric mucosal lesions, and gastric cancer are all well-recognized causes of hematemesis. Esophageal varices, in particular, can cause significant and sometimes life-threatening bleeding.

3-2 (103G18) Which is the endoscopic treatment for early-stage esophageal cancer?

a) Coagulation

b) Mucosal resection

c) Sclerotherapy

d) Stent placement

e) Clipping

 

The correct answer is:

b) Mucosal resection

Explanation:

  • Endoscopic mucosal resection (EMR) is a treatment option for early-stage esophageal cancer that hasn't spread beyond the superficial esophageal tissue. It involves the removal of the affected mucosa and some of the submucosa.

  • Coagulation may be used in endoscopy for controlling bleeding but is not a primary treatment for esophageal cancer.

  • Sclerotherapy is generally used for the treatment of varices, not cancer.

  • Stent placement can be used to alleviate symptoms such as obstruction caused by a tumor but is not a treatment for the cancer itself.

  • Clipping is typically used to close perforations or stop bleeding, not as a treatment for the cancer.

3-3 (105B26) In sub-balloon retrograde transvenous obliteration (BRTO) for gastric varices, which vessel is the balloon catheter inserted into?

a) Short gastric vein

b) Splenic vein

c) Left gastric vein

d) Left renal vein

e) Inferior mesenteric vein

 

The correct answer is:

d) Left renal vein

Explanation:

  • BRTO is primarily used for the management of gastric varices that are often associated with portal hypertension. The procedure involves the occlusion of variceal blood flow to reduce the risk of bleeding. The balloon catheter is typically inserted through the left renal vein because gastric varices often drain into the systemic circulation via the gastrorenal shunt. This shunt commonly connects the gastric varices with the left renal vein, making it an access point for the procedure.

3-4 (106A12) Which of the following is not an exacerbating factor for gastroesophageal reflux disease (GERD)?

a) Recumbent position

b) Nitrate medications

c) High-fat diet

d) Weight loss

e) Calcium channel blockers

 

The correct answer is:

d) Weight loss

Explanation:

  • Weight loss is generally recommended as a management strategy for GERD, as excess weight can increase abdominal pressure and exacerbate reflux symptoms.

  • Recumbent position, nitrate medications, a high-fat diet, and calcium channel blockers can all potentially worsen GERD symptoms. Lying down can lead to reflux, nitrates can relax the lower esophageal sphincter, a high-fat diet can delay gastric emptying, and calcium channel blockers can also relax the lower esophageal sphincter, leading to increased reflux.

3-5 (107D16) What are two typical symptoms of gastroesophageal reflux disease (GERD) in adults?

a) Acid regurgitation

b) Heartburn

c) Weight loss

d) Pain when fasting

e) Hypersalivation

 

The correct answers are:

a) Acid regurgitation

b) Heartburn

Explanation:

  • Acid regurgitation and heartburn are hallmark symptoms of GERD. Acid regurgitation is the sensation of stomach fluid coming up through the throat or mouth, often described as a "wet burp" or even vomiting acid. Heartburn is a burning discomfort felt behind the breastbone, often spreading towards the neck and throat.

  • Weight loss could occur in severe cases of GERD or if the condition leads to eating difficulties due to pain, but it is not typically considered a symptom of GERD.

  • Pain when fasting is less commonly associated with GERD; symptoms of GERD, including heartburn, are often worse after eating, not during fasting.

  • Hypersalivation can occur in response to acid reflux, but it is not as commonly reported as a symptom of GERD compared to acid regurgitation and heartburn.

3-6 (110A12) Which of the following factors found in advanced esophageal cancer indicates that resection, including the primary lesion, is appropriate?

a) Liver metastases

b) Brain metastases

c) Tracheal infiltration

d) Aortic invasion

e) Regional lymph node metastasis

 

The correct answer is:

e) Regional lymph node metastasis

Explanation:

  • Regional lymph node metastasis can still be considered operable in certain cases of esophageal cancer, depending on the extent of spread and the overall health of the patient. Surgery, including resection of the primary lesion and affected lymph nodes, can be part of a curative treatment plan.

  • Liver metastases, brain metastases, tracheal infiltration, and aortic invasion generally indicate more advanced disease and typically make surgical resection with curative intent less feasible due to the spread of cancer beyond the original site and involvement of critical structures or distant organs.

3-7 (111D5) Which organ is most commonly used for reconstruction after subtotal esophagectomy?

a) Stomach

b) Large intestine

c) Small intestine

d) Myocutaneous flap

e) Artificial esophagus

 

The correct answer is:

a) Stomach

Explanation:

  • The stomach is the most commonly used organ for reconstruction after subtotal esophagectomy. It is often fashioned into a tubular shape to replace the resected esophagus, facilitating the continuation of the alimentary tract from the pharynx to the stomach. This method is preferred due to its relative simplicity, the stomach's robust blood supply, and its capacity to mimic the esophagus's function.

3-8 (111D16) Which two conditions can develop suddenly after vomiting?

a) Achalasia of the esophagus

b) Idiopathic esophageal rupture

c) Bochdalek hernia

d) Functional dyspepsia

e) Mallory-Weiss syndrome

 

The correct answers are:

b) Idiopathic esophageal rupture

e) Mallory-Weiss syndrome

Explanation:

  • Idiopathic esophageal rupture, also known as Boerhaave syndrome, can occur suddenly after forceful vomiting, leading to a tear in the esophagus.

  • Mallory-Weiss syndrome involves tears in the mucous membrane at the junction of the stomach and esophagus, which can also occur after severe vomiting.

  • Achalasia of the esophagus is a chronic condition characterized by difficulty swallowing due to the esophagus's inability to relax, not something that develops suddenly after vomiting.

  • Bochdalek hernia is a congenital condition involving a diaphragmatic hernia, not related to acute episodes of vomiting.

  • Functional dyspepsia is a chronic disorder characterized by symptoms of indigestion without a clear cause, not typically an acute condition following vomiting.

3-9 (111G18) Which condition is an indication for endoscopic stenting for esophageal strictures?

a) Esophageal cancer

b) Gastroesophageal reflux disease (GERD)

c) Corrosive esophagitis

d) Esophageal achalasia

e) Gastric tube anastomosis stenosis

 

The correct answer is:

a) Esophageal cancer

Explanation:

  • Esophageal cancer can lead to the development of strictures that obstruct the esophagus, making swallowing difficult. Endoscopic stenting is often used to relieve these symptoms by providing a passage for food and liquids.

  • While stenting can be used in cases of corrosive esophagitis and gastric tube anastomosis stenosis to relieve obstruction, esophageal cancer is a primary condition where stenting is utilized for palliation in advanced cases or as a bridge to further treatment.

  • GERD and esophageal achalasia are not typically treated with stenting. GERD is managed with medications and lifestyle changes, and achalasia treatments focus on methods to relax or bypass the lower esophageal sphincter, such as balloon dilation or surgery, not stenting.

3-10 (112A5) Which of the following statements is true about Mallory-Weiss syndrome?

a) It heals spontaneously.

b) The laceration runs laterally.

c) The lesion extends through the entire wall.

d) It is more common on the side of gastric lordosis.

e) There is also a lesion in the duodenum.

 

The correct answer is:

a) It heals spontaneously.

Explanation:

  • Mallory-Weiss syndrome involves tears in the mucosa at the junction of the stomach and esophagus, typically caused by severe vomiting or retching. These tears often heal spontaneously without the need for surgical intervention.

  • The lacerations are longitudinal, not lateral, making option b incorrect.

  • The lesions in Mallory-Weiss syndrome do not usually extend through the entire wall of the stomach or esophagus; they are superficial, affecting only the mucosa, which makes option c incorrect.

  • There's no specific association with gastric lordosis for the occurrence of Mallory-Weiss tears, making option d incorrect.

  • Mallory-Weiss syndrome involves the gastroesophageal junction, not the duodenum, making option e incorrect.

3-11 (116A7) Which of the following is not a trigger for gastroesophageal reflux disease (GERD)?

a) Obesity

b) Age (being elderly)

c) Kyphosis (referred to here as "tortoise back")

d) Atrophic gastritis

e) Calcium channel blockers (referred to here as "calcium antagonists")

 

The correct answer is:

d) Atrophic gastritis

Explanation:

  • Obesity is a known risk factor for GERD due to increased abdominal pressure which can promote reflux.
  • Age (being elderly) can be associated with a higher risk of GERD, likely due to a combination of factors including decreased esophageal motility and increased incidence of hiatal hernia.
  • Kyphosis ("tortoise back") is not directly recognized as a trigger for GERD, but severe spinal deformities could theoretically affect the anatomy and function of the gastrointestinal tract. However, it's less directly implicated in GERD compared to other options listed.
  • Atrophic gastritis leads to a reduced secretion of stomach acid. Since GERD involves the reflux of stomach acid into the esophagus, conditions that reduce stomach acid are not typical triggers of GERD.
  • Calcium channel blockers (calcium antagonists) can relax the lower esophageal sphincter, which may allow stomach contents to reflux into the esophagus, thus they can be a trigger for GERD.

Therefore, atrophic gastritis is not a trigger for GERD and, in some cases, might even decrease the likelihood of acid reflux symptoms due to reduced acid production.

3-12 (116D5) Which treatment is performed using an upper gastrointestinal endoscope?

a) Esophageal diverticulum

b) Esophageal achalasia

c) Esophageal candidiasis

d) Hiatal hernia

e) Non-erosive gastroesophageal reflux disease

 

The correct answer is:

b) Esophageal achalasia

Explanation:

  • Esophageal achalasia treatment can involve an endoscopic procedure known as peroral endoscopic myotomy (POEM) or endoscopic balloon dilation. These procedures are designed to relieve the symptoms of achalasia by either cutting or stretching the lower esophageal sphincter to improve swallowing.

For the other conditions listed:

  • Esophageal diverticulum might require endoscopic or surgical intervention if symptomatic, but the treatment approach can vary based on the diverticulum's size and location.
  • Esophageal candidiasis is treated with antifungal medications, not typically requiring endoscopic intervention for treatment, though endoscopy might be used for diagnosis.
  • Hiatal hernia treatment usually involves lifestyle changes, medication, or surgery, but not endoscopic treatment. Surgical repair, when necessary, is not typically performed through an endoscope.
  • Non-erosive gastroesophageal reflux disease (NERD) treatment primarily involves lifestyle modifications and pharmacotherapy, such as proton pump inhibitors or H2 receptor antagonists, rather than endoscopic procedures.

Therefore, for the specific question of which treatment is carried out using an upper gastrointestinal endoscope, esophageal achalasia (option b) is the most accurate answer.

Clinical

4-1 (101H46) A 62-year-old man suddenly experienced vomiting blood after dinner and was subsequently admitted to the hospital. He had a history of receiving a blood transfusion due to a traffic injury at the age of 35. Five years ago, he was diagnosed with liver damage during a medical check-up.

Based on the provided information, which is the most appropriate treatment?

a) Administration of a histamine H2 receptor antagonist

b) Spraying with a thrombin solution

c) Injection of adrenaline

d) Endoscopic variceal ligation

e) Hepatic artery embolization

The correct answer is:

d) Endoscopic variceal ligation

Explanation:

 

Given the history of liver damage, which may indicate the presence of cirrhosis, and the acute presentation of hematemesis (vomiting blood), the patient is likely suffering from bleeding esophageal varices. Endoscopic variceal ligation (EVL) is the preferred treatment for acute variceal bleeding as it directly addresses the source of bleeding.

 

  • Administration of a histamine H2 receptor antagonist (a) can help reduce gastric acid secretion and is often used in peptic ulcer disease but would not be the first-line treatment for suspected variceal bleeding.

  • Spraying with a thrombin solution (b) and injection of adrenaline (c) are measures that can be used for temporary hemostasis during endoscopic procedures, but they are adjunctive therapies and not definitive treatments like EVL.

  • Hepatic artery embolization (e) is typically used in the context of hepatocellular carcinoma or other liver tumors, not for acute variceal bleeding.

4-2 (103D56) A 50-year-old man presented with epigastric pain. After consuming a large amount of alcohol, he experienced violent vomiting and was found to have a moderate amount of left pleural effusion on chest X-rays.

Which is the most likely diagnosis?

a) Acute myocardial infarction

b) Mallory-Weiss syndrome

c) Boerhaave syndrome

d) Duodenal ulcer perforation

e) Acute pancreatitis

 

The correct answer is:

c) Boerhaave syndrome

Explanation:

  • Boerhaave syndrome refers to a spontaneous esophageal rupture that typically follows episodes of violent vomiting. The presence of pleural effusion, particularly on the left side, is a common finding due to the leakage of gastric contents into the pleural cavity, which can occur with esophageal rupture.

  • Mallory-Weiss syndrome involves mucosal tears at the gastroesophageal junction also caused by forceful vomiting but typically does not lead to pleural effusion.

  • Acute myocardial infarction (a) can present with epigastric pain but is not directly associated with violent vomiting or pleural effusion as a primary symptom.

  • Duodenal ulcer perforation (d) can cause severe abdominal pain and often leads to free air under the diaphragm seen on X-rays, not typically pleural effusion.

  • Acute pancreatitis (e) can be related to heavy alcohol use and cause epigastric pain, and while it can sometimes be associated with pleural effusions, the history of violent vomiting makes Boerhaave syndrome a more likely diagnosis.

4-3, 4 (104B56, 104B58) A 76-year-old man presented to the hospital with fever and difficulty breathing.

Recent medical history: The patient has experienced dysphagia for 2 months, which was left untreated. Five days ago, he began choking when drinking water. Since yesterday, he has been feeling feverish and short of breath. Additionally, he has lost 8 kg in weight over the last 6 months.

Past medical and family history: There is nothing notable to mention.

Current symptoms: The patient is alert, with a height of 170 cm and a weight of 52 kg. His body temperature is 38.9°C, pulse rate is 104 beats per minute, and blood pressure is 150/88 mmHg. Coarse crackles are audible in the left lower lung field.

Laboratory findings: Urine analysis: protein (-), glucose (-). Blood analysis: red blood cells 3.25 million, hemoglobin 10.1 g/dL, hematocrit 30%, white blood cells 9,800, platelets 370,000. Blood biochemistry: glucose 88 mg/dL, total protein 5.6 g/dL, albumin 2.6 g/dL, creatinine 0.9 mg/dL, total bilirubin 1.0 mg/dL, AST 30 IU/L, ALT 22 IU/L, ALP 198 IU/L (reference range 115–359), amylase 138 IU/L (reference range 37–160).

Which two symptoms are associated with this patient's condition?

a) Jaundice

b) Cough

c) Diarrhea

d) Hoarseness

e) Abdominal distension

 

The two symptoms most likely associated with this patient's condition are:

b) Cough

d) Hoarseness

Explanation:

 

The patient's presentation of fever, difficulty breathing, and history of dysphagia, along with the physical finding of coarse crackles in the lungs, suggest a respiratory issue such as aspiration pneumonia, which can cause a cough and hoarseness.

Which treatment is least appropriate for this patient?

a) Endotracheal stenting

b) Endoscopic mucosal resection

c) Total parenteral nutrition

d) Anti-cancer chemotherapy

e) Gastrostomy

 

The least appropriate treatment for the suspected condition is:

b) Endoscopic mucosal resection

Explanation:

 

Endoscopic mucosal resection is a procedure for removing localized lesions within the gastrointestinal tract. Given the patient's symptoms of respiratory distress and dysphagia, along with weight loss and fever, without evidence of a lesion that requires endoscopic resection, this treatment is not appropriate. The other options could be considered based on the final diagnosis and overall clinical condition.

4-5 (104I45) A 54-year-old man presented to the hospital with complaints of heartburn. Six months ago, he began experiencing heartburn approximately twice a week. Recently, he has noticed that after eating, epigastric pain and a sensation of fullness have emerged, and the heartburn has intensified. He is alert, with a height of 168 cm and a weight of 78 kg. His body temperature is 36.4°C, pulse rate is 72/min, and blood pressure is 122/68 mmHg. There are no abnormalities in heart or breath sounds. The abdomen is flat, soft, and non-tender. Blood analysis reveals red blood cells at 4.88 million, hemoglobin at 13.5 g/dL, hematocrit at 40%, white blood cells at 7,400, and platelets at 280,000. Blood biochemistry shows glucose at 138 mg/dL, creatinine at 0.8 mg/dL, total cholesterol at 248 mg/dL, triglycerides at 125 mg/dL, total bilirubin at 1.0 mg/dL, AST at 28 IU/L, and ALT at 62 IU/L. Endoscopic images of the upper gastrointestinal tract are available for review.

Which treatments are appropriate?

a) Antifungal medication

b) Aspirin

c) Anti-cancer chemotherapy drugs

d) Corticosteroids

e) Proton pump inhibitors

The correct treatment, based on the symptoms presented, is:

e) Proton pump inhibitors

Explanation:

 

Proton pump inhibitors (PPIs) are the standard treatment for heartburn and other symptoms of gastroesophageal reflux disease (GERD), as they effectively reduce the production of stomach acid. The patient's symptoms are consistent with GERD, and PPIs would be used to manage these symptoms and promote healing of the esophageal mucosa if damaged by acid reflux.

The other medications listed are not typically used for the treatment of heartburn:

 

  • Antifungal medication (a) is used for fungal infections, which the patient does not appear to have.
  • Aspirin (b) can actually exacerbate heartburn by irritating the stomach lining.
  • Anti-cancer chemotherapy drugs (c) are used for treating cancer, not heartburn.
  • Corticosteroids (d) are used to treat inflammatory conditions but are not a standard treatment for heartburn and can worsen gastric symptoms in some cases.

4-6 (106I68) A 23-year-old man presented to the hospital with a complaint of vomiting blood. He had consumed more than five cups of Japanese sake at his company's welcome party the previous night, but could not recall the exact amount. Upon waking, he vomited gastric juice and was alarmed to see fresh blood in the vomitus, prompting him to seek medical attention. No abnormalities were noted during the physical examination. At the hospital, he vomited a small amount of fresh blood again, leading to an emergency upper gastrointestinal endoscopy. The endoscopic image of the esophagus is shown below.

Based on the endoscopic image and clinical presentation, which is the appropriate treatment?

a) Clipping

b) Sclerotherapy

c) Mucosal resection

d) Stent placement

e) Hassab's procedure (also known as a portacaval anastomosis)

The appropriate treatment, given the history of acute vomiting of blood following a large intake of alcohol and the endoscopic findings, is:

a) Clipping

Explanation:

 

Endoscopic clipping can be used to mechanically close a bleeding vessel or mucosal tear such as a Mallory-Weiss tear, which is a possible diagnosis in this case given the history of acute alcohol intake followed by retching or vomiting. Clipping is a minimally invasive procedure that can effectively control bleeding in the gastrointestinal tract.

The other treatments listed are less likely to be appropriate given the acute nature of the presentation:

  • Sclerotherapy (b) is typically used for varices, which are not described here.
  • Mucosal resection (c) is used for removing localized lesions and is not indicated for acute bleeding without a defined lesion amenable to resection.
  • Stent placement (d) is used for obstructions or perforations, not for acute bleeding.
  • Hassab's procedure (e) is a surgical intervention for portal hypertension and is not indicated for acute upper gastrointestinal bleeding without evidence of portal hypertension or varices.

4-7 (107G55) A 25-year-old man visited the hospital complaining of vomiting blood. After consuming alcohol with a friend, he experienced vomiting in the bathroom. The initial vomitus contained food residues, but after two or three episodes, blood appeared in the vomitus. His blood pressure was recorded at 72 mmHg. Upon examination, there were no abnormalities in the eyelid conjunctiva, heart, or lung sounds. His abdomen was flat and soft without tenderness, but there was mild enhancement of intestinal sounds. A digital rectal examination showed no abnormalities.

Which diagnostic test is most useful for diagnosis?

a) Abdominal CT

b) Abdominal angiography

c) Upper gastrointestinal angiography

d) Abdominal ultrasonography

e) Upper gastrointestinal endoscopy

 

The most useful diagnostic test for this scenario is:

e) Upper gastrointestinal endoscopy

Explanation:

 

Upper gastrointestinal endoscopy is the most direct and informative test to visualize the upper digestive tract, allowing for the examination of the esophagus, stomach, and duodenum to identify possible sources of bleeding, such as a Mallory-Weiss tear, esophageal varices, or a peptic ulcer. Given the history of acute vomiting and the presence of blood, endoscopy is the preferred initial investigation to diagnose the cause of the hematemesis.

4-8 (113A33) A 38-year-old woman presented to the hospital with a sensation of tightness in her anterior chest. She has been experiencing this sensation when swallowing food for two years. However, her symptoms worsened one month ago, making it difficult for her to consume sufficient food, prompting her to seek medical evaluation. Her blood work is as follows: total protein 7.1 g/dL, albumin 4.2 g/dL, total bilirubin 0.9 mg/dL, AST 22 U/L, ALT 19 U/L, LDH 195 U/L (reference range 176-353), creatinine 0.8 mg/dL, blood glucose 88 mg/dL, sodium 140 mEq/L, potassium 4.3 mEq/L, and chloride 100 mEq/L. An endoscopic image of the upper gastrointestinal tract (A) and an esophageal contrast image (B) are provided.

Which of the following is less likely to be observed in this patient?

a) Aspiration

b) Chest pain

c) Cough

d) Swallowing acid

e) Weight loss

The symptom less likely to be observed in this patient is:

d) Swallowing acid

Explanation:

 

The provided images and symptoms are indicative of a motility disorder, such as achalasia, based on the description of chest tightness and difficulty swallowing. Achalasia is characterized by the inability of the lower esophageal sphincter to relax and the abnormal motility of the esophageal body. This can lead to symptoms such as chest pain (b), weight loss (e) due to difficulty in eating, and potentially cough (c) if food or saliva is aspirated (a) into the airway. However, "swallowing acid" (d), which could be indicative of gastroesophageal reflux disease (GERD), is less likely because achalasia usually prevents stomach contents from re-entering the esophagus due to the tightness of the lower esophageal sphincter.

 

"Swallowing acid" may be a less precise term for acid reflux, which is not commonly associated with achalasia. Instead, patients with achalasia often experience regurgitation of undigested food, not acid, because the food tends to get stuck in the esophagus rather than being refluxed from the stomach.

4-9 (113A44) An 81-year-old man visited the hospital with the primary complaint of difficulty swallowing. He has been aware of this issue for one month, and over the past two weeks, it has progressed to the point where eating has become difficult. He is currently undergoing hormone therapy for prostate cancer. His vital signs are: height 160 cm, weight 56 kg, body temperature 36.1°C, pulse rate 72/min, blood pressure 136/88 mmHg, and respiratory rate 14/min. An endoscopic image of his upper gastrointestinal tract has been provided.

Which condition is most likely indicated by the symptoms and endoscopic image?

a) Barrett's esophagus

b) GERD (Gastroesophageal Reflux Disease)

c) Eosinophilic esophagitis

d) Esophageal achalasia

e) Esophageal candidiasis

The correct answer is:

e) Esophageal candidiasis

Explanation:

  • Esophageal candidiasis is often seen in immunocompromised individuals, such as those undergoing cancer treatments. It is characterized by white plaques along the esophageal lining, which can cause difficulty swallowing or dysphagia.

  • Barrett's esophagus (a) typically presents as a complication of chronic GERD, with a distinctive appearance different from the white plaques associated with candidiasis.

  • Gastroesophageal reflux disease (GERD) (b) usually causes inflammation without the white plaques seen in candidiasis.

  • Eosinophilic esophagitis (c) is associated with allergenic responses and presents with rings and furrows, not white plaques.

  • Esophageal achalasia (d) typically shows a dilated esophagus with retained food and a closed lower sphincter, not the presence of white plaques.

The endoscopic image, which would typically show whitish plaques or lesions in candidiasis, is the key factor in leading to the diagnosis of esophageal candidiasis.

4-10 (114F58) A 62-year-old man was brought to the hospital by ambulance after experiencing an episode of vomiting blood. He suddenly vomited blood in the morning, prompting his family to call for emergency services. Upon examination, his oxygen saturation (SpO2) was 96% with a nasal cannula delivering 3 L/min of oxygen. He had mild anemia in the palpebral conjunctiva and jaundice in the ocular conjunctiva. Abdominal examination revealed distension and fluid waves, indicative of ascites, and there was edema in the lower legs. A rectal examination showed black stool, suggestive of melena. Blood tests showed erythrocytes at 3.28 million, hemoglobin at 9.5 g/dL, hematocrit at 32%, leukocytes at 4,800, and critically low platelets at 40,000. The prothrombin time-international normalized ratio (PT-INR) was elevated at 1.6, indicating a coagulopathy. Biochemical findings revealed hypoalbuminemia and elevated total and direct bilirubin levels, suggesting liver dysfunction. Lactate dehydrogenase (LD) and alkaline phosphatase (ALP) levels were also elevated, and the patient had hyperammonemia, which can be associated with hepatic encephalopathy. The electrolyte levels pointed towards a slightly hyponatremic state. An endoscopic image of the upper gastrointestinal tract is provided.

Which treatments are appropriate? Select two.

a) Ligation

b) Sclerotherapy

c) Stent placement

d) Endoscopic submucosal dissection (ESD)

e) Sengstaken-Blakemore tube placement

The correct answers are:

a) Ligation

b) Sclerotherapy

Explanation:

  • Ligation and sclerotherapy are endoscopic procedures commonly used to control bleeding from esophageal varices, which are likely given the patient's symptoms and signs of liver disease (evidenced by jaundice, ascites, coagulopathy, and hypoalbuminemia).

  • Stent placement (c) is not typically used for variceal bleeding but rather for obstructions or perforations in the gastrointestinal tract.

  • Endoscopic submucosal dissection (d) is a treatment for early-stage gastrointestinal cancers and is not suitable for variceal bleeding.

  • Sengstaken-Blakemore tube placement (e) is a temporary measure to control variceal bleeding when endoscopic interventions are not available or have failed. Given that endoscopic options are viable, this would be less preferable as an initial treatment but can be kept as a backup if the recommended treatments fail.

4-11 (115D53) A 50-year-old man presented to the hospital with complaints of heartburn. He first noticed heartburn after consuming fried foods one year ago, and has experienced persistent, unpleasant heartburn daily for the past two months. His medical and family histories are unremarkable. He has a long history of smoking, with 40 cigarettes per day for 30 years, and drinking, with 3 cups of Japanese sake daily for 30 years. His height is 165 cm, and he weighs 85 kg. An upper gastrointestinal endoscopy image is provided.

Which of the following is applicable to this patient?

a) There are diverticula in the lower esophagus.

b) There is an indication for acid secretion inhibitors.

c) Balloon dilation is required.

d) It is recommended to sleep with your feet up.

e) Esophageal pH monitoring is essential for diagnosis.

The correct answer is:

b) There is an indication for acid secretion inhibitors.

Explanation:

  • The endoscopic image suggests the presence of inflammation or irritation consistent with gastroesophageal reflux disease (GERD), which is commonly treated with acid secretion inhibitors to reduce gastric acid production and alleviate symptoms of heartburn.

  • Diverticula in the lower esophagus (a) would typically be visualized as pouch-like outpouchings, which are not indicated in the provided image.

  • Balloon dilation (c) is a treatment for esophageal strictures or achalasia, which are characterized by difficulty swallowing rather than heartburn.

  • Recommending sleeping with elevated feet (d) is not a standard treatment for heartburn. Instead, elevating the head of the bed is often suggested to prevent nocturnal reflux.

  • Esophageal pH monitoring (e) can be a part of the diagnostic process for GERD, especially when the diagnosis is unclear, but it is not always essential if the clinical presentation and endoscopic findings strongly suggest GERD.

Given the typical presentation of GERD and the significant risk factors including heavy smoking and alcohol consumption, the use of acid secretion inhibitors such as proton pump inhibitors or H2 receptor antagonists is a standard approach to treatment. Lifestyle modifications, including dietary changes and cessation of smoking and alcohol intake, would also be recommended for this patient.

4-12 (116D37) A 67-year-old man presented to the hospital with the chief complaint of dysphagia that started 2 months ago. Over this period, he experienced a weight loss of 6 kg. Endoscopy of the upper gastrointestinal tract and subsequent pathological examination revealed a diagnosis of squamous cell carcinoma located in the upper esophagus of the upper chest. He has received a combination of radiation therapy and chemotherapy.

Which pathology is most likely to have caused pneumonia in this patient?

a) Recurrence of mediastinal lymph nodes

b) Local recurrence of esophageal cancer

c) Tracheoesophageal fistula

d) Esophageal stricture

e) Mediastinitis

 

The correct answer would be:

c) Tracheoesophageal fistula

Explanation:

 

A tracheoesophageal fistula can occur as a complication of esophageal cancer due to tumor invasion or as a late side effect of radiation therapy, leading to an abnormal connection between the trachea and esophagus. This allows for the passage of food and liquids into the respiratory tract, predisposing the patient to aspiration pneumonia.

 

While other options like mediastinal lymph node recurrence (a) and local recurrence of esophageal cancer (b) could contribute to the dysphagia, they are less likely to directly cause pneumonia. An esophageal stricture (d) could also cause dysphagia but would not typically lead to pneumonia unless it was associated with aspiration. Mediastinitis (e) is an inflammation of the mediastinum and could be a result of a tracheoesophageal fistula or an esophageal perforation but is not as directly associated with pneumonia as a tracheoesophageal fistula.

General

5-1 (102G22) For which condition is an upper gastrointestinal barium angiography contraindicated?

a) Achalasia of the esophagus

b) Gastritis

c) Gastric ulcer

d) Perforated duodenal ulcer

e) Duodenal papillary tumor

 

The correct answer is:

d) Perforated duodenal ulcer

 

Explanation:

Upper gastrointestinal barium studies are contraindicated in cases of suspected perforation, such as a perforated duodenal ulcer, due to the risk of barium leaking into the peritoneal cavity, which can cause peritonitis, a severe and potentially life-threatening inflammation. In such cases, a water-soluble contrast agent is preferred for imaging to assess for perforation. Other options listed do not typically present a contraindication for barium studies unless there is an associated risk or presence of perforation.

5-2 (105I25) Which of the following conditions is not an indication for Helicobacter pylori eradication therapy?

a) Gastric ulcer

b) Gastroesophageal reflux disease (GERD)

c) Duodenal ulcer

d) Idiopathic thrombocytopenic purpura (ITP)

e) Gastric MALT lymphoma

 

The correct answer is:

b) Gastroesophageal reflux disease (GERD)

Explanation:

 

Helicobacter pylori eradication therapy is indicated for conditions associated with the bacterium, such as gastric ulcers, duodenal ulcers, idiopathic thrombocytopenic purpura (ITP), and gastric MALT lymphoma because H. pylori infection is implicated in the pathogenesis of these conditions. On the other hand, GERD is not primarily caused by H. pylori infection, and therefore, eradication therapy is not typically indicated solely for GERD. However, there may be cases where H. pylori eradication is considered in GERD patients for other reasons, such as prevention of gastric cancer or peptic ulcers, particularly in areas with a high prevalence of H. pylori infection.

5-3 (106D8) What is type 0-IIc gastric cancer?

a) Localized ulcerative advanced cancer

b) Mass-forming advanced cancer

c) Diffusely infiltrating advanced cancer

d) Superficial depressed early-stage cancer

e) Superficial elevated early-stage cancer

 

The correct answer is:

d) Superficial depressed early-stage cancer

Explanation:

 

Type 0-IIc refers to a superficial depressed type of gastric cancer according to the Japanese classification of gastric carcinoma. This type of cancer is an early-stage gastric cancer characterized by a shallow depression or ulceration. It is not an advanced form of cancer, which would typically present as a more invasive or mass-forming lesion. Instead, it is more localized and involves only the superficial layers of the gastric mucosa.

5-4 (107I10) What is the most appropriate treatment for an active peptic ulcer caused by nonsteroidal anti-inflammatory drugs (NSAIDs)?

a) Antimicrobials

b) Antacids

c) Anticholinergic drugs

d) Corticosteroids

e) Proton pump inhibitors

 

The correct answer is:

e) Proton pump inhibitors

Explanation:

  • Proton pump inhibitors (PPIs) are the treatment of choice for NSAID-induced peptic ulcers because they effectively reduce gastric acid secretion, allowing the ulcer to heal.

  • Antimicrobials (a) are used if there is a co-infection with Helicobacter pylori in peptic ulcer disease.

  • Antacids (b) can provide symptomatic relief by neutralizing stomach acid but do not promote ulcer healing as effectively as PPIs.

  • Anticholinergic drugs (c) reduce acid secretion by blocking the action of acetylcholine on the stomach, but they are not the first line of treatment due to their side effect profile.

  • Corticosteroids (d) are not indicated for peptic ulcer disease and can actually exacerbate ulcers due to their anti-inflammatory effects.

5-5 (107I34) Which conditions are associated with Helicobacter pylori infection? Choose two.

a) Atrophic gastritis

b) Gastric anisakiasis

c) Dumping syndrome

d) Gastric MALT lymphoma

e) Mallory-Weiss syndrome

 

The correct answers are:

a) Atrophic gastritis

d) Gastric MALT lymphoma

Explanation:

  • Atrophic gastritis is a condition where chronic inflammation leads to the thinning and loss of the gastric mucosa. It is commonly caused by a chronic Helicobacter pylori infection, which can damage the stomach lining over time.

  • Gastric MALT lymphoma is a type of cancer originating from the mucosa-associated lymphoid tissue in the stomach. The development of this lymphoma is closely linked to chronic infection with Helicobacter pylori. Treatment for H. pylori often leads to regression of the lymphoma.

Conditions not directly associated with H. pylori infection:

  • Gastric anisakiasis (b) results from ingesting larvae present in raw or undercooked fish, not related to H. pylori.

  • Dumping syndrome (c) occurs after gastric surgery and is not caused by H. pylori infection.

  • Mallory-Weiss syndrome (e) involves tears in the lining of the esophagus or stomach near the gastroesophageal junction, typically following violent vomiting, and is not directly associated with H. pylori infection.

5-6 (108I9) What defines early-stage gastric cancer?

a) There is no indentation.

b) There are no lymph node metastases.

c) The major axis is less than 1 cm.

d) The height of the elevation is less than 5mm.

e) The cancer is confined to the mucosa and submucosa, regardless of lymph node involvement.

 

The correct answer is:

e) The cancer is confined to the mucosa and submucosa, regardless of lymph node involvement.

Explanation:

 

Early-stage gastric cancer is defined by its depth of invasion rather than the presence of lymph node metastases or specific measurements of the tumor's size or elevation. Early gastric cancer specifically refers to tumors that invade the mucosa or submucosa layers of the stomach wall but have not penetrated deeper into the muscularis propria or beyond. This definition includes cancers that may or may not have lymph node involvement, making option e the correct choice. The other options do not accurately describe the criteria for early-stage gastric cancer.

5-7 (110A19) What are the indications for Helicobacter pylori eradication therapy? Choose two.

a) Gastric ulcer

b) Gastric GIST (Gastrointestinal Stromal Tumor)

c) Type 3 gastric cancer

d) Gastroesophageal reflux disease (GERD)

e) Gastric MALT lymphoma

 

The correct answers are:

a) Gastric ulcer

e) Gastric MALT lymphoma

Explanation:

  • Gastric ulcer: Helicobacter pylori infection is a known cause of gastric ulcers. Eradicating the bacteria can promote healing of the ulcer and prevent recurrence.

  • Gastric MALT lymphoma: This type of lymphoma is often associated with H. pylori infection. Eradication of the bacteria can lead to regression of the lymphoma in many cases.

The other options are not directly related to H. pylori eradication therapy:

  • Gastric GIST (b) is a type of tumor that arises from the digestive tract's connective tissue, and its treatment does not typically involve H. pylori eradication.

  • Type 3 gastric cancer (c) refers to a specific morphological type of gastric cancer. While H. pylori is a risk factor for gastric cancer, eradication therapy is not typically a direct treatment for existing gastric cancer.

  • GERD (d) is a condition characterized by acid reflux into the esophagus. While there is some association between H. pylori and GERD, eradication therapy is not a primary treatment for GERD.

5-8 (111I22) Which of the following statements is true regarding the treatment of Helicobacter pylori-positive non-bleeding gastric ulcers?

a) Hospitalization is required.

b) Histamine H2 receptor antagonists are the first line of treatment.

c) Administration of mucosal protective agents is necessary even after successful eradication treatment.

d) Eradication therapy involves intravenous administration of proton pump inhibitors and antimicrobials.

e) Periodic upper gastrointestinal endoscopy is still necessary after successful eradication treatment.

 

The correct answer is:

e) Periodic upper gastrointestinal endoscopy is still necessary after successful eradication treatment.

Explanation:

  • Hospitalization (a) is not typically required for the treatment of H. pylori-positive non-bleeding gastric ulcers unless there are complications or other medical conditions that warrant inpatient care.

  • While histamine H2 receptor antagonists (b) can be used to reduce gastric acid secretion and promote ulcer healing, the primary treatment for H. pylori-positive ulcers involves eradication of the bacterium, typically with a combination of proton pump inhibitors and antimicrobials.

  • Administration of mucosal protective agents (c) may support the healing process, but the necessity for their continued use after successful eradication of H. pylori depends on the individual's clinical scenario and risk factors for ulcer recurrence, not a universal requirement.

  • Eradication therapy (d) for H. pylori typically involves oral administration of a proton pump inhibitor and at least two antimicrobials. Intravenous administration might be considered in specific clinical situations but is not the standard route.

  • Periodic upper gastrointestinal endoscopy (e) is recommended after successful eradication treatment to confirm the healing of the ulcer, monitor for potential complications, and ensure that there is no recurrence of H. pylori infection or development of malignancy, making this statement true.

5-9 (112C11) Which diagnostic tool is useful in diagnosing gastric submucosal tumors?

a) Magnifying endoscope

b) Dye endoscope

c) Ultrasound endoscope

d) Capsule endoscope

e) Double balloon endoscope

 

The correct answer is:

c) Ultrasound endoscope

Explanation:

  • An ultrasound endoscope (or endoscopic ultrasound, EUS) is particularly useful for diagnosing gastric submucosal tumors because it allows for detailed imaging of the layers of the stomach wall. EUS can differentiate between tumors originating in different layers and assess the depth of tumor invasion, making it invaluable for the diagnosis and staging of gastric submucosal tumors.

  • A magnifying endoscope (a) and a dye endoscope (b) can enhance the visualization of the mucosal surface and vascular patterns, which may be helpful in diagnosing mucosal lesions but are less effective for submucosal tumors.

  • A capsule endoscope (d) is used primarily for examining the small intestine and is less effective for detailed analysis of gastric submucosal tumors due to its inability to provide ultrasound imaging.

  • A double balloon endoscope (e) is designed for deep enteroscopy and is useful for visualizing and accessing distant parts of the small bowel rather than for the detailed imaging of submucosal tumors in the stomach.

5-10 (113A7) Which organ is least likely to be invaded by advanced carcinoma of the gastric body?

a) Liver

b) Pancreas

c) Omentum

d) Gallbladder

e) Transverse colon

 

The correct answer is:

d) Gallbladder

 

Explanation:

Advanced carcinoma of the gastric body can metastasize or directly invade adjacent structures due to its anatomical location and the natural progression of the disease. While the liver (a), pancreas (b), omentum (c), and transverse colon (e) are more commonly involved due to direct extension or metastasis, the gallbladder (d) is less frequently invaded by gastric cancer. This is due to the anatomical positioning and the patterns of metastatic spread, making direct invasion of the gallbladder by gastric cancer relatively rare compared to the other listed organs.

Clinical

6-1 (107E52) A 28-year-old man came to the hospital with a complaint of epigastric pain. He began to notice pain in the epigastric region 10 days ago. The pain often appeared on an empty stomach and was reduced after eating. There is nothing special to note in his medical history. His height is 168 cm, weight 56 kg, body temperature 36.2°C, pulse 64/min, and blood pressure 122/62 mmHg. There are no abnormalities in the eyelid or ocular conjunctiva. Heart and breathing sounds are normal. The abdomen is flat and soft, and the liver and spleen are not palpable. No abnormalities are observed in intestinal murmurs. Urinary findings: protein (-), sugar (-). Blood findings: erythrocytes 4.6 million, Hb 13.9 g/dL, Ht 44%, leukocytes 8,300, platelets 240,000. Blood biochemical findings: albumin 4.1 g/dL, urea nitrogen 18 mg/dL, creatinine 0.8 mg/dL, total bilirubin 0.9 mg/dL, AST 22 IU/L, ALT 32 IU/L, LD 286 IU/L (standard 176-353), ALP 221 IU/L (standard 115-359), Na 136 mEq/L, K 4.2 mEq/L, Cl 102 mEq/L. Which tests are needed to determine the course of treatment?

a) Fecal occult blood reaction

b) Protein leakage test

c) Urea breath test

d) Serum CEA measurement

e) Tuberculin reaction

 

The correct answer is:

c) Urea breath test

Explanation:

  • The urea breath test is a non-invasive diagnostic procedure used to detect Helicobacter pylori (H. pylori) infections. H. pylori is a common cause of peptic ulcers, which often present with epigastric pain, especially when the stomach is empty. The pain typically improves after eating, which can neutralize stomach acid.

  • A fecal occult blood test (a) would be used to detect hidden blood in the stool, which is common with gastrointestinal bleeding, but there is no indication of bleeding in this patient's presentation.

  • The protein leakage test (b) is not standard for epigastric pain and is more relevant to gastrointestinal protein-losing enteropathy conditions.

  • Serum CEA measurement (d) is a tumor marker primarily used to monitor colorectal cancer and is not indicated in this scenario.

  • The tuberculin test (e) is used to diagnose tuberculosis infection and is unrelated to the current symptoms.

Given the patient's symptoms, the urea breath test would be the most appropriate next step in determining the cause of his epigastric pain and guiding treatment, which could include the eradication of H. pylori if present.

6-2 (108E45) A 75-year-old man presented to the hospital complaining of epigastric pain. For three months, he had experienced a persistent and gradually worsening dull pain in the epigastric area, which was now associated with a loss of appetite. On physical examination, his abdomen was soft and flat, with a palpable, non-mobile mass around the umbilicus. No lymphadenopathy was detectable in the cervical, axillary, or inguinal regions. Images from an upper gastrointestinal endoscopy (A) and an abdominal contrast-enhanced CT scan (B) are provided below.

What is the degree of progression (stage) according to the TNM classification?

a) IA

b) IB

c) II

d) III

e) IV

The correct answer is:

e) IV

Explanation:

  • The TNM classification system is used to describe the extent of cancer's spread. 'T' describes the size of the original (primary) tumor and whether it has invaded nearby tissue, 'N' describes nearby (regional) lymph nodes that are involved, and 'M' describes distant metastasis (spread of cancer to other parts of the body).

  • Given the patient's symptoms of persistent epigastric pain, appetite loss, and the palpable mass, as well as the implied findings from the endoscopic and CT images, stage IV cancer is indicated. This stage is characterized by the spread of cancer beyond the stomach to other parts of the body, which could be inferred from the non-mobile mass, suggesting possible invasion into other tissues or organs. Accurate staging would typically involve more detailed imaging results and pathological confirmation through biopsy or surgical exploration.

6-3, 4, 5 (108G67, 108G68, 108G69) A 62-year-old man presented to the hospital with a chief complaint of epigastric pain. Three months ago, he began to experience occasional epigastric discomfort. Recently, a colleague was diagnosed with gastric cancer after presenting with similar symptoms and subsequently underwent surgery, which raised his concerns about possibly having stomach cancer himself. He also noticed a decrease in appetite, so he self-medicated with over-the-counter stomach medication, which seemed to improve the discomfort and his appetite. Despite this, the epigastric discomfort persisted. A week ago, he sustained a lower back bruise, for which he received treatment at a local clinic and recovered within two days. Over the past three days, the epigastric pain has intensified, even waking him up at night. Concerned by media reports linking Helicobacter pylori infection to stomach cancer, he requested an upper gastrointestinal endoscopy, which was performed.

Past medical history: Unremarkable. Social history: No history of smoking. Current symptoms: The patient is alert, with a body temperature of 36.7°C, a pulse of 76 beats per minute, blood pressure of 128/70 mmHg, and a respiratory rate of 16 breaths per minute. The abdomen is flat, and there is no palpable liver or spleen enlargement. Mild tenderness in the epigastric area is noted, but no palpable mass is found. Laboratory findings: Urinalysis is negative for protein, glucose, and occult blood. Blood tests reveal a red blood cell count of 3.3 million, hemoglobin of 11.8 g/dL, hematocrit of 32%, a white blood cell count of 7,200, and platelets of 240,000.

Which questions should I add?

a) "Do you eat raw fish?"

b) "Are you taking painkillers?"

c) "Haven't you been abroad lately?"

d) "Have you been drinking well water lately?"

e) "Do you have colon cancer in your blood-related family?"

 

The appropriate question to add is:

(B) "Are you taking painkillers?"

Explanation:

  • Asking about raw fish (A) is relevant for parasitic infections.
  • Inquiring about painkillers (B) is pertinent because nonsteroidal anti-inflammatory drugs (NSAIDs) can cause gastritis and peptic ulcers, potentially leading to epigastric pain.
  • Recent travel (C) could be associated with infectious diseases.
  • Well water (D) can be a source of contaminants but is less likely to be related to gastric symptoms unless contaminated with H. pylori.
  • Family history of colon cancer (E) may be relevant for a genetic predisposition to cancer but is not directly related to H. pylori infection or epigastric pain.

Helicobacter pylori infection was diagnosed, and the results are shown below. Which diagnostic method was used?

a) Culture method

b) Blood antibody measurement

c) Stool antigen measurement

d) Urea breath test

e) Urease test

 

The diagnostic method used is:

(E) Urease test

Explanation:

  • The image shows a rapid urease test, which is commonly used to detect H. pylori. The presence of the bacteria is indicated by a color change in the medium due to the urease enzyme produced by H. pylori converting urea into ammonia and carbon dioxide, altering the pH.

Which is the appropriate treatment?

a) Blood transfusion

b) Endoscopic hemostasis

c) Single-agent administration of antimicrobials

d) Endoscopic submucosal dissection

e) Administration of proton pump inhibitors

 

The appropriate treatment is:

(E) Administration of proton pump inhibitors

Explanation:

  • Proton pump inhibitors (PPIs) are a cornerstone of treatment for conditions related to gastric acid, including peptic ulcer disease caused by H. pylori. They help to reduce stomach acid, protect the stomach lining, and create an environment in which antibiotics can be more effective against H. pylori.
  • Endoscopic hemostasis (B) would be used if there was active bleeding, which is not indicated here.
  • Blood transfusion (A) is for significant blood loss.
  • Single-agent antimicrobials (C) are not sufficient for H. pylori eradication, which typically requires a combination therapy.
  • Endoscopic submucosal dissection (D) is a treatment for early gastric cancer, not for H. pylori infection.

6-6 (108I47) A 57-year-old man presented to the hospital with palpitations. He had a pyloric gastrectomy six months prior due to early-stage gastric cancer. Post-surgery, he was mindful to eat small meals, which helped improve his physical condition and increase his appetite over time. Three months ago, he started eating more than before. Two months ago, he began experiencing episodes of nausea, cold sweats, and palpitations, occurring 2-3 hours post-meal. These episodes would last about 30-40 minutes and then subside. Snacking at the onset of symptoms seemed to provide relief. There was no exercise-induced chest pain or postprandial heartburn, yet concern about these symptoms led him to seek medical advice. His medical history was notable only for early gastric cancer. His current vital signs showed a pulse rate of 72 beats/min and a blood pressure of 138/72 mmHg. Cardiac and respiratory examinations were unremarkable. The abdomen was flat, soft, and non-tender.

What is the most likely diagnosis?

a) Angina pectoris

b) Pernicious anemia

c) Atrial fibrillation

d) Gastroesophageal reflux disease (GERD)

e) Dumping syndrome

 

The correct answer is:

e) Dumping syndrome

Explanation:

 

Dumping syndrome is a condition that can occur after surgery to remove all or part of the stomach or after surgery to bypass the stomach to help lose weight. Symptoms often occur 10 to 30 minutes after eating and can include palpitations, lightheadedness, nausea, and sweating, which align with the patient's description. These symptoms can be attributed to the rapid emptying of stomach contents into the small intestine, which can cause a sudden shift in body fluid into the gut and result in the cardiovascular symptoms described. The fact that the symptoms appear 2-3 hours after eating and are relieved by snacking suggests the late phase of dumping syndrome, where the rapid absorption of sugars leads to a spike in insulin levels, causing hypoglycemia and corresponding symptoms.

  • Angina pectoris (a) typically presents as chest pain or discomfort that may radiate to the arm, neck, or jaw and is often exercise-induced, which does not align with the patient's symptoms.

  • Pernicious anemia (b) is a cause of vitamin B12 deficiency which can lead to anemia and neurological symptoms, but it is not typically associated with food intake or the symptoms described.

  • Atrial fibrillation (c) is an irregular and often rapid heart rate that can lead to palpitations, but it is not typically related to food intake.

  • GERD (d) could cause heartburn after eating but would not explain the palpitations, cold sweats, or relationship to the timing of symptoms post-meal.

6-7 (109D34) A 46-year-old man presented for a detailed examination after a medical checkup two weeks ago revealed a Helicobacter pylori infection through a blood test.

Which one should be done next?

a) Eradication treatment

b) Urea breath test

c) Blood pepsinogen measurement

d) Upper gastrointestinal endoscopy

e) Helicobacter pylori antigen measurement in stool

 

The correct answer is:

d) Upper gastrointestinal endoscopy

Explanation:

  • Upper gastrointestinal endoscopy is the appropriate choice for a closer examination following a positive test for Helicobacter pylori infection. It allows direct visualization of the stomach and duodenum, biopsy of suspicious areas, and assessment for the presence of ulcers, gastritis, or cancer, which can be associated with H. pylori infection.

  • Eradication treatment (a) may be indicated after confirming the diagnosis and assessing for ulcer disease or other complications. It is generally not initiated solely on the basis of a blood test without further diagnostic evaluation.

  • The urea breath test (b) and Helicobacter pylori antigen measurement in stool (e) are non-invasive tests used to diagnose active H. pylori infection. However, since there's already an indication of infection, endoscopy would provide more comprehensive information.

  • Blood pepsinogen measurement (c) is a marker of gastric function and can indicate atrophic gastritis, which is a risk factor for gastric cancer. It may be used in conjunction with other tests but is not a direct test for H. pylori infection.

6-8 (110A30) A 75-year-old man presented to the hospital with complaints of a loss of appetite and abdominal bloating. He had a history of undergoing a total gastrectomy for advanced gastric cancer one year prior and received adjuvant chemotherapy postoperatively. However, he had not followed up at the hospital since. Over the past three weeks, he gradually noticed a decreased appetite and increasing bloating, which notably worsened in the last two days. Despite reduced food intake, he experienced constipation and fatigue. His physical examination revealed a height of 167 cm, a weight of 45 kg, a body temperature of 36.6°C, a pulse of 84/min, and blood pressure of 136/80 mmHg. There was mild pallor in the ocular conjunctiva. No palmar erythema or spider angiomas were noted. The abdomen was distended and tense, with a flattened umbilicus and the presence of fluid waves, suggesting ascites. There was no palpable hepatosplenomegaly. Lower extremity edema was also present. A plain abdominal CT scan is provided below.

Given this clinical scenario, which test should be performed next?

a) PET/CT

b) Ascites cytology

c) Abdominal plain MRI

d) Upper gastrointestinal endoscopy

e) Lower gastrointestinal endoscopy

The appropriate test to perform next would be:

b) Ascites cytology

Explanation:

  • Ascites cytology is the examination of the fluid accumulated in the abdomen (ascites) to detect the presence of malignant cells, which could indicate peritoneal carcinomatosis, commonly associated with advanced or recurrent gastric cancer.

  • PET/CT (a) could be used to search for metastatic disease, but given the acute presentation with ascites, cytology would provide quicker and more direct evidence of possible peritoneal spread.

  • Abdominal plain MRI (c) could give more detail about soft tissue structures but is less specific for detecting and characterizing ascites compared to cytology.

  • Upper gastrointestinal endoscopy (d) is not immediately indicated unless there are signs suggesting an upper GI source for the symptoms such as bleeding or obstruction.

  • Lower gastrointestinal endoscopy (e) is less likely to provide useful information in the setting of a history of gastric cancer and current symptoms of ascites and bloating.

6-9 (110A36) A 62-year-old man visited the hospital for a routine follow-up after undergoing a gastrectomy. Three months prior, he received a pyloric gastrectomy with Billroth I reconstruction for stage Ib gastric cancer and has since been attending monthly check-ups. His oral intake has improved gradually. Recently, he has been experiencing general malaise, cold sweats, and hand tremors a few hours after eating, occurring 3 to 4 times per week. His current measurements are a height of 173 cm and a weight of 63 kg, with a body temperature of 36.7°C, a pulse of 80/min, and blood pressure of 132/82 mmHg. His abdomen is flat and soft without any palpable masses.

Considering these symptoms, which of the following are possible causes?

a) Anemia

b) Dehydration

c) Undernutrition

d) Hypoglycemia

e) Hyponatremia

 

The likely cause of the symptoms, given the history of gastrectomy and the timing of symptoms postprandially, would be:

d) Hypoglycemia

Explanation:

  • Hypoglycemia can occur after a gastrectomy as a part of a condition known as "dumping syndrome," where rapid gastric emptying can lead to early postprandial hyperglycemia followed by reactive hypoglycemia.

  • Anemia (a) may be a concern post-gastrectomy due to changes in absorption, but it typically does not cause acute symptoms of malaise, cold sweat, and tremors in relation to food intake.

  • Dehydration (b) would not be expected to occur in a pattern related to meal times.

  • Undernutrition (c) could contribute to the patient's symptoms but would not typically cause acute symptoms tied to eating.

  • Hyponatremia (e) typically presents with neurological symptoms and is less likely to be related to the timing of food intake.

6-10, 11, 12 (110G60, 110G61, 110G62) A 78-year-old man presented at the hospital for the evaluation and management of gastric lesions. He had experienced epigastric pain for the past two years. Following an upper gastrointestinal contrast study at a local clinic, an abnormality was detected, and he was referred for further assessment.

Past Medical History: He has been receiving treatment for hypertension for the last decade.

Social History: He has a long history of smoking, approximately 30 cigarettes per day for 58 years, and moderate alcohol consumption, with two cups of Japanese sake daily for the same duration. He is independent in his activities of daily living (ADLs).

Family History: His father had stomach cancer.

Clinical Findings: The patient is alert and oriented, with a height of 167 cm and a weight of 48 kg. His vital signs include a body temperature of 36.4°C, a pulse rate of 68 beats per minute, a blood pressure of 130/80 mmHg, and a respiratory rate of 18 breaths per minute. The ocular and palpebral conjunctivae are normal. No lymphadenopathy is palpable, and auscultation reveals no abnormalities in cardiac or respiratory sounds. The abdomen is flat with epigastric tenderness, but no palpable masses are present.

Laboratory Results: Hematologic values show a red blood cell count of 3.1 million/µL, hemoglobin of 10.6 g/dL, hematocrit of 29%, a white blood cell count of 8,600/µL, and a platelet count of 220,000/µL. Biochemical analysis reveals a total protein level of 6.8 g/dL, albumin of 3.0 g/dL, total bilirubin of 0.9 mg/dL, AST of 30 IU/L, ALT of 42 IU/L, LDH of 350 IU/L, ALP of 242 IU/L, γ-GTP of 83 IU/L, amylase of 84 IU/L, creatine kinase (CK) of 130 IU/L, blood urea nitrogen (BUN) of 5.0 mg/dL, fasting blood glucose of 108 mg/dL, HbA1c of 5.8%, total cholesterol of 57 mg/dL, cholinesterase of 300 IU/L, sodium of 140 mEq/L, potassium of 3.8 mEq/L, chloride of 100 mEq/L, and C-reactive protein (CRP) of 0.4 mg/dL. Arterial blood gas on room air shows a pH of 7.39, PaCO2 of 50 Torr, PaO2 of 75 Torr, and HCO3- of 29 mEq/L. Pulmonary function tests reveal a %VC of 68% and %FEV1 of 54%.

Imaging: The patient provided an upper gastrointestinal contrast image (A), and further endoscopic examination (B) was performed, both of which are presented below.

Which is the gross type of the upper gastrointestinal endoscopic image?

a) Type 0

b) Type 1

c) Type 2

d) Type 3

e) Type 4

 

The correct answer would be:

e) Type 4

Explanation:

 

The endoscopic image typically associated with Type 4 gastric cancer, also known as linitis plastica, shows a diffusely infiltrating cancer that thickens the stomach wall and often leads to a reduction in the stomach's capacity and flexibility. This type of cancer does not form a discrete mass and can cause the stomach to appear rigid.

 

Which interventions are necessary for preoperative management? Select three.

a) Smoking cessation

b) Blood transfusion

c) Nutritional management

d) Albumin supplementation

c) Respiratory rehabilitation

 

The correct answers would be:

a) Smoking cessation

c) Nutritional management

e) Respiratory rehabilitation

Explanation:

 

  • Non-smoking: Smoking cessation is important to reduce the risk of surgical complications and promote better wound healing.
  • Nutrition Management: Proper nutrition is critical to ensure the patient is in optimal condition for surgery and recovery.
  • Respiratory Rehabilitation: Given the patient's history of smoking and advanced age, respiratory rehabilitation may be important to improve lung function and reduce the risk of postoperative pulmonary complications.

These interventions are aimed at improving the patient's overall health status and mitigating risks associated with surgery. Smoking cessation is universally recommended. Nutrition management ensures the patient has adequate energy and resources for healing. Respiratory rehabilitation is particularly relevant given the long history of smoking, which can impact lung capacity and function, crucial for recovery from major surgery like a gastrectomy.

 

Which is the appropriate surgical procedure for gastric lesions?

a) Total gastrectomy

b) Local excision

c) Fundectomy

d) Pyloroplasty

e) Pancreaticoduodenectomy

 

The correct answer would be:

a) Total gastrectomy

Explanation:

 

Given the advanced age of the patient, the potential for widespread involvement of the stomach by the cancer, and the possibility of Type 4 gastric cancer, a total gastrectomy might be considered to fully remove the cancerous tissue. The decision would ultimately depend on the patient's overall health, the extent of the disease, and a thorough evaluation by the surgical and oncology team.

6-13 (110I65) A 71-year-old woman visited the hospital after a submucosal tumor was detected during a routine medical check-up. Six months prior, a comprehensive health screening revealed a mobile submucosal tumor approximately 3.0 cm in diameter located in the body of the stomach. Abdominal CT scans did not indicate any lesions in other organs. Hematoxylin and eosin (H-E) stained sections of the tumor tissue (A, B), along with immunohistochemical staining for KIT (C) and CD34 (D), are provided.

Which is the most appropriate treatment?

a) Total gastrectomy

b) Radiation therapy

c) Anti-cancer chemotherapy

d) Partial gastrectomy

e) Helicobacter pylori eradication treatment

 

The correct answer is:

d) Partial gastrectomy

Explanation:

  • Partial gastrectomy is the most appropriate treatment for a gastrointestinal stromal tumor (GIST), which is likely in this scenario given the presence of a submucosal tumor in the stomach with good mobility, and especially if immunohistochemical staining is positive for KIT (CD117) and CD34, which are commonly expressed in GISTs.

  • Total gastrectomy (a) involves removing the entire stomach and is typically reserved for more extensive disease.

  • Radiation therapy (b) is not a standard treatment for primary GISTs, as these tumors are often resistant to radiation.

  • Anti-cancer chemotherapy (c) with drugs like imatinib may be used in cases where the GIST is metastatic or unresectable, or as an adjuvant treatment following surgery depending on the risk stratification.

  • Helicobacter pylori eradication treatment (e) is used for conditions associated with Helicobacter pylori infection, such as peptic ulcers or gastric MALT lymphoma, and is not relevant for a GIST.

The choice between a partial and total gastrectomy would ultimately depend on the tumor's location, size, and other factors evaluated intraoperatively and preoperatively. In cases where the tumor is localized and with no metastasis, a partial gastrectomy to remove the affected part of the stomach is often curative.

6-14, 15 (111B060, 111B061) A 63-year-old man was brought to the hospital by ambulance due to vomiting blood. He has been experiencing epigastric pain after eating for the past two years but did not seek medical care due to his busy schedule. Recently, he has consistently felt epigastric discomfort, loss of appetite, and general malaise. He noticed his stool was black two days ago and vomited a significant amount of bright red blood this morning after defecation, prompting him to call an ambulance.

His medical history includes a cerebral infarction at around age 50, for which he has been prescribed antiplatelet drugs. He has smoked 20 cigarettes per day for 43 years and consumed one cup of soju per day for 38 years. His father had stomach cancer, and his mother has hypertension.

Upon examination, he is conscious, with a height of 168 cm, weight of 72 kg, body temperature of 36.3°C, heart rate of 96/min, blood pressure of 98/68 mmHg, and respiratory rate of 20/min. His SpO2 is 96% on room air. The palpebral conjunctiva is anemic, but there is no jaundice in the ocular conjunctiva. Cardiac and pulmonary auscultation reveals no abnormalities. The abdomen is flat and soft with mild epigastric tenderness.

Laboratory tests reveal erythrocytes at 2.76 million, hemoglobin at 8.7 g/dL, hematocrit at 28%, white blood cells at 6,400, and platelets at 280,000. Biochemical tests show total protein at 6.8 g/dL, albumin at 3.4 g/dL, total bilirubin at 0.8 mg/dL, AST at 45 U/L, ALT at 56 U/L, LDH at 344 U/L, ALP at 322 U/L, γ-GTP at 87 U/L, amylase at 120 U/L, creatine kinase at 78 U/L, blood urea nitrogen at 28 mg/dL, creatinine at 0.8 mg/dL, uric acid at 6.3 mg/dL, blood glucose at 108 mg/dL, HbA1c at 5.8%, total cholesterol at 250 mg/dL, triglycerides at 190 mg/dL, sodium at 140 mEq/L, potassium at 4.2 mEq/L, and chloride at 100 mEq/L.

Which is the appropriate treatment?

a) Regionalized gastric resection

b) Pyloric gastrectomy

c) Endoscopic hemostasis

d) Elective vagus nerve resection

e) Endoscopic submucosal dissection

 

The correct answer is:

c) Endoscopic hemostasis

Explanation:

 

Endoscopic hemostasis directly addresses the source of bleeding, which can be vital in stabilizing the patient. Techniques might include clipping, banding, or injection of hemostatic agents.

 

Which is appropriate as a subsequent treatment?

a) Digestive enzyme drug

b) Gastric mucosal protectors

c) Medications to improve motor function

d) Proteolytic enzyme inhibitors

e) Proton pump inhibitors

 

The correct answer is:

e) Proton pump inhibitors

Explanation:

 

Proton pump inhibitors would be the best choice. They reduce stomach acid, helping to heal the bleeding site and prevent further episodes.

6-16 (113F65) A 52-year-old man underwent a comprehensive medical checkup, during which a 2 cm stomach cancer was discovered in the gastric vestibule through an upper gastrointestinal endoscopy.

Which tools are useful for determining the course of treatment? Choose two.

a) Magnifying endoscope

b) Transnasal endoscopy

c) Ultrasound endoscope

d) Capsule endoscope

e) Balloon endoscope

 

Correct Answers:

a) Magnifying endoscope

c) Ultrasound endoscope

Explanation:

  • Magnifying endoscope provides detailed visualization of the mucosal layer, allowing for the assessment of the lesion's characteristics, such as its surface pattern and microvascular architecture. This detailed view can help determine the malignancy's nature and its potential invasiveness, crucial for planning the treatment approach.

  • Ultrasound endoscope (EUS) is essential for staging gastric cancer, as it can assess the depth of tumor invasion into the gastric wall and detect lymph node involvement. Accurate staging is critical for deciding on the most appropriate treatment, such as surgery, chemotherapy, or a combination thereof.

  • Transnasal endoscopy (b), while less invasive, primarily offers diagnostic views similar to standard endoscopy without the added benefit of depth assessment provided by EUS.

  • Capsule endoscope (d) and Balloon endoscope (e) are innovative tools for exploring the small intestine and are less suited for detailed examination and staging of gastric cancer, which requires the capabilities of magnifying and ultrasound endoscopes.

6-17 (114A72) A 61-year-old man was taken to the hospital by ambulance with epigastric pain. Seven days ago, he was prescribed NSAIDs at a clinic near his home because he was experiencing back pain. This morning, he suddenly developed epigastric pain, which rapidly intensified, prompting him to call an ambulance. Upon admission, his SpO2 was 98% with oxygen administration via a nasal cannula at 3 L/min. Physical examination revealed a flat abdomen with tenderness and mild rebound pain in the epigastric area. Laboratory findings revealed an erythrocyte count of 4.83 million, Hb 15.7 g/dL, Ht 47%, leukocyte count of 14,700, and platelets at 300,000. Biochemical findings indicated a total protein of 7.3 g/dL, albumin 4.2 g/dL, and total bilirubin 0.9 mg/dL. Enzyme levels were AST 20 U/L, ALT 15 U/L, LD 170 U/L, ALP 265 U/L, γ-GT 66 U/L, and amylase 52 U/L. Kidney function tests showed urea nitrogen at 17 mg/dL and creatinine at 1.0 mg/dL. A simple CT scan of the abdomen showed no abnormalities, with CRP at 2.2 mg/dL.

Which is the appropriate response? Choose three.

a) Antimicrobial administration

b) Nasogastric tube placement

c) Hyperbaric oxygen therapy

d) Abdominal angiography

e) Proton pump inhibitor administration

Correct Answers:

a) Antimicrobial administration

b) Nasogastric tube placement

e) Proton pump inhibitor administration

Explanation:

 

  • Antimicrobial administration may be considered for acute epigastric pain. There is a possibility of an ulcer with a risk of perforation, which could necessitate antimicrobials.

     

  • Nasogastric tube placement can help relieve gastric distension and remove gastric contents, which is useful if there is concern for an ulcer or gastritis.

     

  • Proton pump inhibitor administration decreases gastric acid secretion, which is beneficial if an NSAID-induced peptic ulcer is suspected.

     

  • Hyperbaric oxygen therapy (c) is typically not indicated for epigastric pain or suspected peptic ulcers. Abdominal angiography (d) might be considered if there was a suspicion of gastrointestinal bleeding, but there is no evidence from the presented case to support this intervention.

6-18 (114F70) A 76-year-old woman visited the hospital for stomach cancer treatment. After a histopathological examination, a healing resection was diagnosed, and the healing of the post-resection ulcer was confirmed by upper gastrointestinal endoscopy.

Doctor: "① I will prescribe NSAIDs and a triple therapy regimen of antibiotics to eradicate H. pylori. ② You should take these medications three times a day—morning, noon, and evening—for one month. Side effects may include diarrhea and skin rashes, but ③ it's important to continue the medication even if you experience these side effects."

Patient: "I understand."

Doctor: "④ Successful eradication of H. pylori significantly reduces the risk of developing stomach cancer, but ⑤ it is advisable to undergo stomach endoscopy once every one to two years for surveillance."

Patient: "Understood."

Doctor: "We will follow up in two months to check if the eradication has been successful."

In the dialogue, which underlined part is the most appropriate?

a) ①

b) ②

c) ③

d) ④

e) ⑤

 

In the dialogue, the underlined part that is most appropriate is:

e) ⑤

Explanation:

  • Part ① might not be appropriate if NSAIDs are to be avoided in a patient with a history of stomach cancer and post-resection, given the risk of gastric irritation or ulcer reformation.
  • Part ② is incorrect as antibiotics for H. pylori eradication are typically taken for 7-14 days, not one month.
  • Part ③ is problematic because while patients are often advised to continue medication despite mild side effects, they should report severe side effects to their doctor immediately and not merely "be patient."
  • Part ④ is misleading because while H. pylori eradication reduces the risk of certain types of stomach cancer, it does not guarantee that cancer will not occur.
  • Part ⑤ is the most appropriate, as regular surveillance with endoscopy is recommended for patients with a history of stomach cancer to monitor for recurrence or the development of new lesions.

6-19 (116A68) A 38-year-old woman presented at the hospital complaining of abdominal pain. She had not had a bowel movement for the past week and experienced symptoms of vomiting, abdominal pain, and distension, which progressively worsened. Her SpO2 was 97% while breathing room air. Examination revealed abdominal distension with hyperactive bowel sounds, and a rectal exam indicated induration in the recto-uterine pouch (Douglas's pouch). An abdominal CT scan showed dilation of the bowel from the ileum to the rectum and a small amount of ascites in the pelvic cavity. Upper gastrointestinal endoscopy identified type 4 gastric cancer.

Which are the appropriate actions to take? Choose two.

a) Total gastrectomy

b) Gastrostomy

c) Swallowing training

d) Nasogastric tube placement

e) Central venous nutrition

 

Given the clinical presentation and findings, the correct course of action for a 38-year-old woman with signs of intestinal obstruction and diagnosis of type 4 gastric cancer would likely include:

d) Nasogastric tube placement

e) Central venous nutrition

Explanation:

  • Nasogastric tube placement is often used in the management of intestinal obstruction to decompress the stomach, relieve symptoms of vomiting and abdominal distention, and reduce the risk of aspiration. It can provide immediate relief for symptoms caused by the obstruction.

  • Central venous nutrition may be necessary to ensure the patient receives adequate nutrition, especially if there is a significant obstruction that impairs the ability to eat and digest food normally. Since the patient has not had a bowel movement for a week and presents with vomiting and abdominal pain, enteral nutrition might not be feasible.

  • Total gastrectomy (a) could be considered as part of the treatment for gastric cancer, but it is not the immediate step when dealing with an acute obstruction and the patient's current unstable condition. A thorough preoperative assessment and planning would be required.

  • Gastrostomy (b) can be used for long-term enteral feeding if the lower digestive tract is functional, but given the current acute scenario, it is not an immediate action.

  • Swallowing training (c) would not be applicable in this case as the primary issue does not seem to be related to the mechanics of swallowing, but rather to the obstruction caused by the cancer and possibly the intestinal dilatation.

The immediate priority is to stabilize the patient, manage the obstruction, and maintain nutrition and hydration. Definitive treatment for the gastric cancer, such as surgery, would be considered after the acute issue is addressed and the patient's overall condition allows for it.

6-20 (116A72) A 40-year-old man presented at the hospital complaining of epigastric pain that began yesterday. His SpO2 was 97% on room air, he was fully conscious, his abdomen was flat but showed signs of peritonitis, such as muscular defense throughout, with tenderness and rebound pain in the upper abdomen. Intestinal sounds were absent. His blood work showed a red blood cell count of 4.88 million, hemoglobin 14.8 g/dL, hematocrit 44%, white blood cells at 12,200, and platelets at 330,000. His ALT was 19 U/L, LDH 135 U/L (standard 120-245), blood urea nitrogen 10 mg/dL, creatinine 0.7 mg/dL, and C-reactive protein 1.7 mg/dL. Simple abdominal CT images were taken, showing conditions of the soft tissue (A) and the pulmonary field (B).

Which actions should be taken first? Choose three.

a) Fasting

b) Infusion

c) Laparocentesis

d) Nasogastric tube insertion

e) Upper gastrointestinal endoscopy

Given this scenario, the appropriate immediate interventions would likely include:

a) Fasting

b) Infusion

d) Nasogastric tube insertion

Explanation:

  • Fasting can prevent further stimulation of the gastrointestinal tract, which could exacerbate pain or increase the risk of aspiration in the case of surgery.
  • Infusion can ensure hydration and electrolyte balance, especially if the patient is unable to take oral fluids due to fasting or if there has been vomiting or dehydration.
  • Nasogastric tube insertion decompress the stomach, relieve nausea and vomiting, and prevent further gastrointestinal content from leaking if there is a perforation.

These interventions are part of the initial management of a patient with a suspected surgical abdomen to stabilize the patient before further diagnostic procedures or surgical intervention.

  • Laparocentesis (c) and Upper gastrointestinal endoscopy (e) are also diagnostic and potentially therapeutic interventions, but they are not typically the first line of action in an emergency setting without a more definitive diagnosis. Laparocentesis (or diagnostic peritoneal lavage) might be considered if there's suspicion of intra-abdominal bleeding or infection, and endoscopy might be used subsequently to identify the source of gastrointestinal bleeding or perforation after initial stabilization. However, given the acute presentation and the need for immediate stabilization, the first three actions (fasting, infusion, nasogastric tube insertion) are most critical.